Week 1

Ace your homework & exams now with Quizwiz!

Compressibility factor

Ratio of the experimentally measured molar volume for a gas to its molar volume as computed from the ideal gas equation

Excess reactant

Reactant present in an amount greater than required by the reaction stoichiometry

Neutralization reaction

Reaction between an acid and a base to produce salt and water

Oxidation reduction reaction

Reaction involving a change in oxidation number for one or more reactant elements

Oxidation number

The charge each atom of an element would have in a compound if the compound were ionic

Strong base

Base that reacts completely when dissolved in water to yield hydroxide ions

Dichromate

Cr2O7 -2

Chromate

CrO4 -2

Copper (I)

Cu+

Determine the percent ammonia, NH3, in Co(NH3)6Cl3, to three significant figures.

% NH3 = 38.2%

Calculate the following to four significant figures: (a) the percent composition of ammonia, NH3 (b) the percent composition of photographic fixer solution ("hypo"), Na2S2O3 (c) the percent of calcium ion in Ca3(PO4)2

(a) % N = 82.24%, % H = 17.76%; (b) % Na = 29.08%, % S = 40.56%, % O = 30.36%; (c) % Ca2+ = 38.76%

Silver

Ag+

Aluminum

Al+3

Fluoride

F-

Sulfide

S -2

Zinc

Zn+2

What is the molarity of the diluted solution when each of the following solutions is diluted to the given final volume? (a) 1.00 L of a 0.250-Msolution of Fe(NO3)3 is diluted to a final volume of 2.00 L (b) 0.5000 L of a 0.1222-Msolution of C3H7OH is diluted to a final volume of 1.250 L (c) 2.35 L of a 0.350-Msolution of H3PO4 is diluted to a final volume of 4.00 L (d) 22.50 mL of a 0.025-Msolution of C12H22O11 is diluted to 100.0 mL

(a) 0.125M; (b) 0.04888M; (c) 0.206M; (d) 0.0056M

Round off each of the following numbers to two significant figures: (a) 0.436 (b) 9.000 (c) 27.2 (d) 135 (e) 1.497 × 10^−3 (f) 0.445

(a) 0.44; (b) 9.0; (c) 27; (d) 140; (e) 1.5 × 10−3; (f) 0.44

Determine the molarity for each of the following solutions: (a) 0.444 mol of CoCl2 in 0.654 L of solution (b) 98.0 g of phosphoric acid, H3PO4, in 1.00 L of solution (c) 0.2074 g of calcium hydroxide, Ca(OH)2, in 40.00 mL of solution (d) 10.5 kg of Na2SO4·10H2O in 18.60 L of solution (e) 7.0 × 10−3 mol of I2 in 100.0 mL of solution (f) 1.8 × 104 mg of HCl in 0.075 L of solution

(a) 0.679M; (b) 1.00M; (c) 0.06998M; (d) 1.75M; (e) 0.070M; (f) 6.6

Determine the mass of each of the following: (a) 0.0146 mol KOH (b) 10.2 mol ethane, C2H6 (c) 1.6 × 10^−3 mol Na2 SO4 (d) 6.854 × 10^3 mol glucose, C6 H12 O6 (e) 2.86 mol Co(NH3)6Cl3

(a) 0.819 g; (b) 307 g; (c) 0.23 g; (d) 1.235 × 10^6 g (1235 kg); (e) 765 g

Write conversion factors (as ratios) for the number of: (a) yards in 1 meter (b) liters in 1 liquid quart (c) pounds in 1 kilogram

(a) 1.0936 yd / 1 m ; (b) 0.94635 L / 1 qt ; (c) 2.2046 lb / 1 kg

Use scientific (exponential) notation to express the following quantities in terms of the SI base units in Table 1.3: (a) 0.13 g (b) 232 Gg (c) 5.23 pm (d) 86.3 mg (e) 37.6 cm (f) 54 μm (g) 1 Ts (h) 27 ps (i) 0.15 mK

(a) 1.3 × 10^−4 kg; (b) 2.32 × 10^8 kg; (c) 5.23 × 10^−12 m; (d) 8.63 × 10^−5 kg; (e) 3.76 × 10^−1 m; (f) 5.4 × 10^−5 m; (g) 1 × 10^12 s; (h) 2.7 × 10^−11 s; (i) 1.5 × 10^−4 K

The effect of chlorofluorocarbons (such as CCl2F2) on the depletion of the ozone layer is well known. The use of substitutes, such as CH3CH2F(g), for the chlorofluorocarbons, has largely corrected the problem. Calculate the volume occupied by 10.0 g of each of these compounds at STP: (a) CCl2F2(g) (b) CH3CH2F(g)

(a) 1.85 L CCl2F2; (b) 4.66 L CH3CH2F

Consider this scenario and answer the following questions: On a mid-August day in the northeastern United States, the following information appeared in the local newspaper: atmospheric pressure at sea level 29.97 in. Hg, 1013.9 mbar. (a) What was the pressure in kPa? (b) The pressure near the seacoast in the northeastern United States is usually reported near 30.0 in. Hg. During a hurricane, the pressure may fall to near 28.0 in. Hg. Calculate the drop in pressure in torr.

(a) 101.5 kPa; (b) 51 torr drop

Write a symbol for each of the following neutral isotopes. Include the atomic number and mass number for each. (a) the alkali metal with 11 protons and a mass number of 23 (b) the noble gas element with 75 neutrons in its nucleus and 54 electrons in the neutral atom (c) the isotope with 33 protons and 40 neutrons in its nucleus (d) the alkaline earth metal with 88 electrons and 138 neutrons

(a) 1123Na; (b) 54129Xe; (c) 3373As; (d) 88226Ra

What is the total mass (amu) of carbon in each of the following molecules? (a) CH4 (b) CHCl3 (c) C12H10O6 (d) CH3CH2CH2CH2CH3

(a) 12.01 amu; (b) 12.01 amu; (c) 144.12 amu; (d) 60.05 amu

Calculate the molecular or formula mass of each of the following: (a) P4 (b) H2O (c) Ca(NO3)2 (d) CH3CO2H (acetic acid) (e) C12H22O11 (sucrose, cane sugar)

(a) 123.896 amu; (b) 18.015 amu; (c) 164.086 amu; (d) 60.052 amu; (e) 342.297 amu

Write the symbol for each of the following ions: (a) the ion with a 1+ charge, atomic number 55, and mass number 133 (b) the ion with 54 electrons, 53 protons, and 74 neutrons (c) the ion with atomic number 15, mass number 31, and a 3− charge (d) the ion with 24 electrons, 30 neutrons, and a 3+ charge

(a) 133Cs+; (b) 127I−; (c) 31P3−; (d) 57Co3

Consider the following questions: (a) What is the total volume of the CO2(g) and H2O(g) at 600 °C and 0.888 atm produced by the combustion of 1.00 L of C2H6(g) measured at STP? (b) What is the partial pressure of H2O in the product gases?

(a) 18.0 L; (b) 0.533 atm

Calculate the molar mass of each of the following: (a) the anesthetic halothane, C2HBrClF3 (b) the herbicide paraquat, C12H14N2Cl2 (c) caffeine, C8H10N4O2 (d) urea, CO(NH2)2 (e) a typical soap, C17H35CO2Na

(a) 197.382 g mol^−1; (b) 257.163 g mol^−1; (c) 194.193 g mol^−1; (d) 60.056 g mol^−1; (e) 306.464 g mol^−1

Perform the following calculations and report each answer with the correct number of significant figures. (a) 628 × 342 (b) (5.63 × 10^2) × (7.4 × 10^3) (c) 28.0 / 13.483 (d) 8119 × 0.000023 (e) 14.98 + 27,340 + 84.7593 (f) 42.7 + 0.259

(a) 2.15 × 105; (b) 4.2 × 106; (c) 2.08; (d) 0.19; (e) 27,440; (f) 43.0

Yeast converts glucose to ethanol and carbon dioxide during anaerobic fermentation as depicted in the simple chemical equation here: glucose ⟶ ethanol + carbon dioxide (a) If 200.0 g of glucose is fully converted, what will be the total mass of ethanol and carbon dioxide produced? (b) If the fermentation is carried out in an open container, would you expect the mass of the container and contents after fermentation to be less than, greater than, or the same as the mass of the container and contents before fermentation? Explain. (c) If 97.7 g of carbon dioxide is produced, what mass of ethanol is produced?

(a) 200.0 g; (b) The mass of the container and contents would decrease as carbon dioxide is a gaseous product and would leave the container. (c) 102.3 g

Calculate the molar mass of each of the following: (a) S8 (b) C5H12 (c) Sc2(SO4)3 (d) CH3COCH3 (acetone) (e) C6H12O6 (glucose)

(a) 256.528 g/mol; (b) 72.150 g mol^−1; (c) 378.103 g mol^−1; (d) 58.080 g mol^−1; (e) 180.158 g mol^−1

The pressure of a sample of gas is measured at sea level with a closed-end manometer. The liquid in the manometer is mercury. Determine the pressure of the gas in: (a) torr (b) Pa (c) bar

(a) 264 torr; (b) 35,200 Pa; (c) 0.352 bar

Complete and balance the following oxidation-reduction reactions, which give the highest possible oxidation state for the oxidized atoms. (a) Al(s)+F2(g) ⟶ (b) Al(s)+CuBr2(aq) ⟶ (single displacement) (c) P4(s)+O2(g) ⟶ (d) Ca(s)+H2O(l) ⟶ (products are a strong base and a diatomic gas)

(a) 2Al(s)+3F2(g) ⟶ 2AlF3(s); (b) 2Al(s)+3CuBr2(aq) ⟶ 3Cu(s)+2AlBr3(aq); (c) P4(s)+5O2(g) ⟶ P4O10(s); (d) Ca(s)+2H2O(l) ⟶ Ca(OH)2(aq)+H2(g)

Complete and balance the following acid-base equations: (a) HCl gas reacts with solid Ca(OH)2(s). (b) A solution of Sr(OH)2 is added to a solution of HNO3.

(a) 2HCl(g)+Ca(OH)2(s) ⟶ CaCl2(s)+2H2O(l); (b) Sr(OH)2(aq)+2HNO3(aq) ⟶ Sr(NO3)2(aq)+2H2O(l)

From the balanced molecular equations, write the complete ionic and net ionic equations for the following: (a) K2C2O4(aq)+Ba(OH)2(aq) ⟶ 2KOH(aq)+BaC2O4(s) (b) Pb(NO3)2(aq)+H2SO4(aq) ⟶ PbSO4(s)+2HNO3(aq) (c) CaCO3(s)+H2SO4(aq) ⟶ CaSO4(s)+CO2(g)+H2O(l)

(a) 2K+(aq)+C2O42−(aq)+Ba2+(aq)+2OH−(aq) ⟶ 2K+(aq)+2OH−(aq)+BaC2O4(s) (complete) Ba2+(aq)+C2O42−(aq) ⟶ BaC2O4(s) (net) (b) Pb2+(aq)+2NO3−(aq)+2H+(aq)+SO42−(aq) ⟶ PbSO4(s)+2H+(aq)+2NO3−(aq) (complete) Pb2+(aq)+SO42−(aq) ⟶ PbSO4(s) (net) (c) CaCO3(s)+2H+(aq)+SO42−(aq) ⟶ CaSO4(s)+CO2(g)+H2O(l) (complete) CaCO3(s)+2H+(aq)+SO42−(aq) ⟶ CaSO4(s)+CO2(g)+H2O(l) (net)

Give the number of protons, electrons, and neutrons in neutral atoms of each of the following isotopes: (a) 73Li (b) 12552Te (c) 10947Ag (d) 157N (e) 3115P

(a) 3 protons, 3 electrons, 4 neutrons; (b) 52 protons, 52 electrons, 73 neutrons; (c) 47 protons, 47 electrons, 62 neutrons; (d) 7 protons, 7 electrons, 8 neutrons; (e) 15 protons, 15 electrons, 16 neutrons

Solve these problems about lumber dimensions. (a) To describe to a European how houses are constructed in the US, the dimensions of "two-by-four" lumber must be converted into metric units. The thickness × width × length dimensions are 1.50 in. × 3.50 in. × 8.00 ft in the US. What are the dimensions in cm × cm × m? (b) This lumber can be used as vertical studs, which are typically placed 16.0 in. apart. What is that distance in centimeters?

(a) 3.81 cm × 8.89 cm × 2.44 m; (b) 40.6 cm

Calculate the number of moles and the mass of the solute in each of the following solutions: (a) 2.00 L of 18.5MH2SO4, concentrated sulfuric acid (b) 100.0 mL of 3.8 × 10^−5 MNaCN, the minimum lethal concentration of sodium cyanide in blood serum (c) 5.50 L of 13.3MH2CO, the formaldehyde used to "fix" tissue samples (d) 325 mL of 1.8 × 10^−6 MFeSO4, the minimum concentration of iron sulfate detectable by taste in drinking water

(a) 37.0 mol H2SO4, 3.63 × 10^3 g H2SO4; (b) 3.8 × 10^−6 mol NaCN, 1.9 × 10^−4 g NaCN; (c) 73.2 mol H2CO, 2.20 kg H2CO; (d) 5.9 × 10^−7 mol FeSO4, 8.9 × 10^−5 g FeSO4

Make the conversion indicated in each of the following: (a) the length of a soccer field, 120 m (three significant figures), to feet (b) the height of Mt. Kilimanjaro, at 19,565 ft, the highest mountain in Africa, to kilometers (c) the area of an 8.5- × 11-inch sheet of paper in cm^2 (d) the displacement volume of an automobile engine, 161 in.^3, to liters (e) the estimated mass of the atmosphere, 5.6 × 10^15 tons, to kilograms (f) the mass of a bushel of rye, 32.0 lb, to kilograms (g) the mass of a 5.00-grain aspirin tablet to milligrams (1 grain = 0.00229 oz)

(a) 394 ft; (b) 5.9634 km; (c) 6.0 × 102; (d) 2.64 L; (e) 5.1 × 1018 kg; (f) 14.5 kg; (g) 324 mg

Calculate the molarity of each of the following solutions: (a) 0.195 g of cholesterol, C27H46O, in 0.100 L of serum, the average concentration of cholesterol in human serum (b) 4.25 g of NH3 in 0.500 L of solution, the concentration of NH3 in household ammonia (c) 1.49 kg of isopropyl alcohol, C3H7OH, in 2.50 L of solution, the concentration of isopropyl alcohol in rubbing alcohol

(a) 5.04 × 10^−3 M; (b) 0.499M; (c) 9.92M; (d) 1.1 × 10^−3 (d) 0.029 g of I2 in 0.100 L of solution, the solubility of I2 in water at 20 °C

Calculate these volumes. What is the volume of 25 g iodine, density = 4.93 g/cm^3? What is the volume of 3.28 g gaseous hydrogen, density = 0.089 g/L?

(a) 5.1 mL; (b) 37 L

Silver is often extracted from ores such as K[Ag(CN)2] and then recovered by the reaction 2K|Ag(CN)2|(aq)+Zn(s) ⟶ 2Ag(s)+Zn(CN)2(aq)+2KCN(aq) (a) How many molecules of Zn(CN)2 are produced by the reaction of 35.27 g of K[Ag(CN)2]? (b) What mass of Zn(CN)2 is produced?

(a) 5.337 × 1022 molecules; (b) 10.41 g Zn(CN)2

Determine the molecular mass of the following compounds:

(a) 56.107 amu; (b) 54.091 amu; (c) 199.9976 amu; (d) 97.9950 amu

The pressure of a sample of gas is measured at sea level with an open-end mercury manometer. Assuming atmospheric pressure is 760.0 mm Hg, determine the pressure of the gas in: (a) mm Hg (b) atm (c) kPa

(a) 623 mm Hg; (b) 0.820 atm; (c) 83.1 kP

How many grams of gas are present in each of the following cases? (a) 0.100 L of CO2 at 307 torr and 26 °C (b) 8.75 L of C2H4, at 378.3 kPa and 483 K (c) 221 mL of Ar at 0.23 torr and -54 °C

(a) 7.24 × 10-2 g; (b) 23.1 g; (c) 1.5 × 10-4 g

Calculate these masses. (a) What is the mass of 6.00 cm^3 of mercury, density = 13.5939 g/cm^3? (b) What is the mass of 25.0 mL octane, density = 0.702 g/cm^3?

(a) 81.6 g; (b) 17.6 g

Determine the mass in grams of each of the following: (a) 0.600 mol of oxygen atoms (b) 0.600 mol of oxygen molecules, O2 (c) 0.600 mol of ozone molecules, O3

(a) 9.60 g; (b) 19.2 g; (c) 28.8 g

Determine the mass of each of the following: (a) 2.345 mol LiCl (b) 0.0872 mol acetylene, C2H2 (c) 3.3 × 10^−2 mol Na2 CO3 (d) 1.23 × 10^3 mol fructose, C6 H12 O6 (e) 0.5758 mol FeSO4(H2O)7

(a) 99.41 g; (b) 2.27 g; (c) 3.5 g; (d) 222 kg; (e) 160.1 g

When elemental iron corrodes it combines with oxygen in the air to ultimately form red brown iron(III) oxide called rust. (a) If a shiny iron nail with an initial mass of 23.2 g is weighed after being coated in a layer of rust, would you expect the mass to have increased, decreased, or remained the same? Explain. (b) If the mass of the iron nail increases to 24.1 g, what mass of oxygen combined with the iron?

(a) Increased as it would have combined with oxygen in the air thus increasing the amount of matter and therefore the mass. (b) 0.9 g

The following are properties of isotopes of two elements that are essential in our diet. Determine the number of protons, neutrons and electrons in each and name them. (a) atomic number 26, mass number 58, charge of 2+ (b) atomic number 53, mass number 127, charge of 1−

(a) Iron, 26 protons, 24 electrons, and 32 neutrons; (b) iodine, 53 protons, 54 electrons, and 74 neutrons

The following ionic compounds are found in common household products. Write the formulas for each compound: (a) potassium phosphate (b) copper(II) sulfate (c) calcium chloride (d) titanium(IV) oxide (e) ammonium nitrate (f) sodium bisulfate (the common name for sodium hydrogen sulfate)

(a) K3PO4; (b) CuSO4; (c) CaCl2; (d) TiO2; (e) NH4NO3; (f) NaHSO4

Using the periodic table, identify the lightest member of each of the following groups: (a) noble gases (b) alkaline earth metals (c) alkali metals (d) chalcogens

(a) He; (b) Be; (c) Li; (d) O

Complete and balance the equations for the following acid-base neutralization reactions. If water is used as a solvent, write the reactants and products as aqueous ions. In some cases, there may be more than one correct answer, depending on the amounts of reactants used. (a) Mg(OH)2(s)+HClO4(aq) ⟶ (b) SO3(g)+H2O(l) ⟶ (assume an excess of water and that the product dissolves) (c) SrO(s)+H2SO4(l) ⟶

(a) Mg(OH)2(s)+2HClO4(aq) ⟶ Mg2+(aq)+2ClO4−(aq)+2H2O(l); (b) SO3(g)+2H2O(l) ⟶ H3O+(aq)+HSO4−(aq), (a solution of H2SO4); (c) SrO(s)+H2SO4(l) ⟶ SrSO4(s)+H2O

Classify each of the following as an element, a compound, or a mixture: (a) iron (b) oxygen (c) mercury oxide (d) pancake syrup (e) carbon dioxide (f) a substance composed of molecules each of which contains one hydrogen atom and one chlorine atom (g) baking soda (h) baking powder

(a) element; (b) element; (c) compound; (d) mixture; (e) compound; (f) compound; (g) compound; (h) mixture

Gallium chloride is formed by the reaction of 2.6 L of a 1.44 M solution of HCl according to the following equation: 2Ga+6HCl ⟶ 2GaCl3+3H2. (a) Outline the steps necessary to determine the number of moles and mass of gallium chloride. (b) Perform the calculations outlined.

(a) volume HCl solution ⟶ mol HCl ⟶ mol GaCl3; (b) 1.25 mol GaCl3, 2.2 × 10^2 g GaCl3

A balloon with a volume of 100.21 L at 21 °C and 0.981 atm is released and just barely clears the top of Mount Crumpet in British Columbia. If the final volume of the balloon is 144.53 L at a temperature of 5.24 °C, what is the pressure experienced by the balloon as it clears Mount Crumpet?

0.644 atm

A typical barometric pressure in Denver, Colorado, is 615 mm Hg. What is this pressure in atmospheres and kilopascals?

0.809 atm; 82.0 kPa

A certain nut crunch cereal contains 11.0 grams of sugar (sucrose, C12H22O11) per serving size of 60.0 grams. How many servings of this cereal must be eaten to consume 0.0278 moles of sugar?

0.865 servings, or about 1 serving.

Canadian tire pressure gauges are marked in units of kilopascals. What reading on such a gauge corresponds to 32 psi?

2.2 × 102 kPa

Copper(I) iodide (CuI) is often added to table salt as a dietary source of iodine. How many moles of CuI are contained in 1.00 lb (454 g) of table salt containing 0.0100% CuI by mass?

2.38 × 10−4 mol

Which of the following represents the least number of molecules? (a) 20.0 g of H2O (18.02 g/mol) (b) 77.0 g of CH4 (16.06 g/mol) (c) 68.0 g of CaH2 (42.09 g/mol) (d) 100.0 g of N2O (44.02 g/mol) (e) 84.0 g of HF (20.01 g/mol)

20.0 g H2O represents the least number of molecules since it has the least number of moles.

A weather balloon contains 8.80 moles of helium at a pressure of 0.992 atm and a temperature of 25 °C at ground level. What is the volume of the balloon under these conditions?

217 L

A sample of gallium bromide, GaBr3, weighing 0.165 g was dissolved in water and treated with silver nitrate, AgNO3, resulting in the precipitation of 0.299 g AgBr. Use these data to compute the %Ga (by mass) GaBr3.

22.4%

Standard conditions of temperature and pressure (STP)

273.15 K and 1 atm

Lithium hydroxide may be used to absorb carbon dioxide in enclosed environments, such as manned spacecraft and submarines. Write an equation for the reaction that involves 2 mol of LiOH per 1 mol of CO2. (Hint: Water is one of the products.)

2LiOH(aq)+CO2(g) ⟶ Li2CO3(aq)+H2O(l)

Great Lakes Chemical Company produces bromine, Br2, from bromide salts such as NaBr, in Arkansas brine by treating the brine with chlorine gas. Write a balanced equation for the reaction of NaBr with Cl2.

2NaBr(aq)+Cl2(g) ⟶ 2NaCl(aq)+Br2(l)

The Cullinan diamond was the largest natural diamond ever found (January 25, 1905). It weighed 3104 carats (1 carat = 200 mg). How many carbon atoms were present in the stone?

3.113 × 10^25 C atoms

Titration of a 20.0-mL sample of acid rain required 1.7 mL of 0.0811MNaOH to reach the end point. If we assume that the acidity of the rain is due to the presence of sulfuric acid, what was the concentration of sulfuric acid in this sample of rain?

3.4 × 10^−3 MH2SO4

A spray can is used until it is empty except for the propellant gas, which has a pressure of 1344 torr at 23 °C. If the can is thrown into a fire (T = 475 °C), what will be the pressure in the hot can?

3.40 × 103 torr

How many milliliters of a 0.1500-Msolution of KOH will be required to titrate 40.00 mL of a 0.0656-M solution of H3PO4? H3PO4(aq)+2KOH(aq) ⟶ K2HPO4(aq)+2H2O(l)

34.99 mL KOH

How many milliliters of a soft drink are contained in a 12.0-oz can?

355 mL

Gasoline is sold by the liter in many countries. How many liters are required to fill a 12.0-gal gas tank?

45.4 L

A 20.0-L cylinder containing 11.34 kg of butane, C4H10, was opened to the atmosphere. Calculate the mass of the gas remaining in the cylinder if it were opened and the gas escaped until the pressure in the cylinder was equal to the atmospheric pressure, 0.983 atm, and a temperature of 27 °C.

46.4 g

What volume of 0.600MHCl is required to react completely with 2.50 g of sodium hydrogen carbonate? NaHCO3(aq)+HCl(aq) ⟶ NaCl(aq)+CO2(g)+H2O(l)

49.6 mL

A chemistry student is 159 cm tall and weighs 45.8 kg. What is her height in inches and weight in pounds?

62.6 in (about 5 ft 3 in.) and 101 lb

Soccer is played with a round ball having a circumference between 27 and 28 in. and a weight between 14 and 16 oz. What are these specifications in units of centimeters and grams?

68-71 cm; 400-450 g

What is the molar mass of a gas if 0.281 g of the gas occupies a volume of 125 mL at a temperature 126 °C and a pressure of 777 torr?

72.0 g mol^−1

A sample of carbon monoxide was collected over water at a total pressure of 756 torr and a temperature of 18 °C. What is the pressure of the carbon monoxide? (See Table 9.2for the vapor pressure of water.)

740 torr

Average atomic masses listed by IUPAC are based on a study of experimental results. Bromine has two isotopes, 79Br and 81Br, whose masses (78.9183 and 80.9163 amu, respectively) and abundances (50.69% and 49.31%, respectively) were determined in earlier experiments. Calculate the average atomic mass of bromine based on these experiments.

79.90 amu

The diameter of a red blood cell is about 3 × 10^−4 in. What is its diameter in centimeters?

8 × 10−4 cm

How many moles of gaseous boron trifluoride, BF3, are contained in a 4.3410-L bulb at 788.0 K if the pressure is 1.220 atm? How many grams of BF3?

8.190 × 10-2 mol; 5.553 g

The density of a certain gaseous fluoride of phosphorus is 3.93 g/L at STP. Calculate the molar mass of this fluoride and determine its molecular formula.

88.1 g mol−1; PF3

In a common medical laboratory determination of the concentration of free chloride ion in blood serum, a serum sample is titrated with a Hg(NO3)2 solution. 2Cl−(aq)+Hg(NO3)2(aq) ⟶ 2NO3−(aq)+HgCl2(s) What is the Cl− concentration in a 0.25-mL sample of normal serum that requires 1.46 mL of 8.25 × 10−4 M Hg(NO3)2(aq) to reach the end point?

9.6 × 10−3 MCl−

In Canada and the United Kingdom, devices that measure blood glucose levels provide a reading in millimoles per liter. If a measurement of 5.3 mMis observed, what is the concentration of glucose (C6H12O6) in mg/dL?

95 mg/dL

The density (d) of a substance is an intensive property that is defined as the ratio of its mass (m) to its volume (V). density = mass/volume, d = m/V. Considering that mass and volume are both extensive properties, explain why their ratio, density, is intensive.

Being extensive properties, both mass and volume are directly proportional to the amount of substance under study. Dividing one extensive property by another will in effect "cancel" this dependence on amount, yielding a ratio that is independent of amount (an intensive property).

Phosphate

PO4 -3

What is Boyle's law

PV = constant at constant T and n; If the gas volume is decreased, the container wall area decreases and the molecule wall collision frequency increases, both of which increases the pressure exerted by the gas

Lead (II)

Pb+2

Lead (IV)

Pb+4

How do you calculate percent by mass

Percent by mass = (mass of solute / mass of solution) * 100

How do you calculate percent yield?

Percent yield = (actual yield / theoretical yield) * 100

A student isolated 25 g of a compound following a procedure that would theoretically yield 81 g. What was his percent yield?

Percent yield=31%

Percent composition

Percentage by mass of the various elements in a compound

Explain how you could experimentally determine whether the outside temperature is higher or lower than 0 °C (32 °F) without using a thermometer.

Place a glass of water outside. It will freeze if the temperature is below 0 °C.

Oxyanion

Polyatomic anion composed of a central atom bonded to oxygen atoms

Cation

Positively charged atom or molecule (contains fewer electrons than protons)

Alpha particle

Positively charged particle consisting of two protons and two neutrons

Proton

Positively charged, subatomic particle located in the nucleus; relatively heavy particles with a charge of 1+ and a mass of 1.0073 amu

Three categories of important reactions

Precipitation; Acid base; Oxidation reduction (redox)

Hydrostatic pressure

Pressure exerted by a fluid due to gravity

Partial pressure

Pressure exerted by an individual gas in a mixture

Vapor pressure of water

Pressure exerted by water vapor in equilibrium with liquid water in a closed container at a specific temperature

Amontons's Law

Pressure of a given number of moles of gas is directly proportional to its kelvin temperature when the volume is held constant; The pressure of a given amount of gas is directly proportional to its absolute temperature, provided that the volume does not change

Rounding

Procedure used to ensure that calculated results properly reflect the uncertainty in the measurements used in the calculation

Oxidation

Process in which an element's oxidation number is increased by loss of electrons

Dilution

Process of adding solvent to a solution in order to lower the concentration of solutes

Chemical change

Produce one or more types of matter that differ from the present before the change

Periodic law

Properties of the elements are periodic function of their atomic numbers

What does a nucleus contain?

Protons and neutrons

Compounds

Pure substances that can be broken down by chemical changes

Elements

Pure substances that cannot be broken down into simpler substances by chemical changes

Concentrated

Qualitative term for a solution containing solute at a relatively high concentration

Dilute

Qualitative term for a solution containing solute at a relatively low concentration

Titration analysis

Quantitative chemical analysis method involves measuring the volume of a reactant solution required to completely react with the analyte in a sample

Gravimetric analysis

Quantitative chemical analysis method involving the separation of an analyte from a sample by a physical or chemical process and subsequent mass measurements of the analyte, reaction product, and/or sample

Concentration

Quantitative measure of the relative amounts of solute and solvent present in a solution

Graham's law of effusion

Rates of diffusion and effusion of gases are inversely proportional to the square roots of their molecular masses; Rates of effusion of gases are inversely proportional to the square roots of their densities or to the square roots of their atoms/molecules masses

Parts per billion (ppb)

Ratio of Solute to solution mass multiplied by 109

Stoichiometric factor

Ratio of coefficients in a balanced chemical equation, used in computations relating amounts of reactants and products

Unit conversion factor

Ratio of equivalent quantities expressed with different units; Used to convert from one unit to a different unit

Density

Ratio of mass to volume for a substance or object

Mass volume percent

Ratio of solute mass to solution volume, expressed as a percentage

Mass percentage

Ratio of solute to solution mass expressed as a percentage

Parts per million (ppm)

Ratio of solute to solution mass multiplied by 106

Volume percentage

Ratio of solute to solution volume expressed as a percentage

Rubidium

Rb+

Limiting reactant

Reactant present in an amount lower than required by the reaction stoichiometry, thus limiting the amount of product generated

Acid base reaction

Reaction involving the transfer of a hydrogen ion between reactant species

Precipitation reaction

Reaction that produces one or more insoluble products; When reactants are ionic compounds, sometimes called double displacement or metathesis

Single displacement reaction

Redox reaction involving the oxidation of an elemental substance by an ionic species

Ideal gas law

Relation between the pressure, volume, amount, and temperature of a gas under conditions derived by combination of the simple gas laws; PV = nRT

Stoichiometry

Relationships between the amounts of reactants and products of a chemical reaction

Solid

Rigid and possess a definite shape

Thiosulfate

S2O3 -2

Thiocyanate

SCN -

For which of the following gases should the correction for the molecular volume be largest: CO, CO2, H2, He, NH3, SF6

SF6

Pascal (Pa)

SI unit of pressure; 1 Pa = 1 N/m2

Kelvin

SI unit of temperature; 273.15 K = 0°C

Second

SI unit of time

Cubic meter

SI unit of volume

Sulfate

SO4 -2

Sulfite

SO4 -2

Factor label method

Same thing as a dimensional analysis

Selenide

Se -2

Dalton's atomic theory

Set of postulates that established the fundamental properties of atoms

Structural formula

Shows the atoms in a molecule and how they are connected

Carborundum is silicon carbide, SiC, a very hard material used as an abrasive on sandpaper and in other applications. It is prepared by the reaction of pure sand, SiO2, with carbon at high temperature. Carbon monoxide, CO, is the other product of this reaction. Write the balanced equation for the reaction, and calculate how much SiO2 is required to produce 3.00 kg of SiC.

SiO2+3C ⟶ SiC+2CO, 4.50 kg SiO2

Microscopic domain

Smaller, only really observable through microscopes or more powerful instruments

Atom

Smallest particle of an element that has the properties of that element and can enter into a chemical combination

Tin (II)

Sn+2

Tin (IV)

Sn+4

What did Soddy demonstrate?

Soddy demonstrated that atoms of the same element can differ in mass

Solute

Solution component present in a concentration less than that of the solvent

Solvent

Solution component present in a concentration that is higher relative to other components

Titrant

Solution containing a known concentration of substance that will react with the analyte in a titration analysis

Aqueous solution

Solution for which water is the solvent

Periodic table

Table of the elements that places elements with similar chemical properties close together

Gas

Takes both the shape and volume of its container

Absolute zero

Temperature at which the volume of a gas would be zero according to Charles's law

Outline the steps needed to determine the limiting reactant when 0.50 mol of Cr and 0.75 mol of H3PO4 react according to the following chemical equation. 2Cr+2H3PO4 ⟶ 2CrPO4+3H2 Determine the limiting reactant.

The conversion needed is mol Cr ⟶ molH3PO4. Then compare the amount of Cr to the amount of acid present. Cr is the limiting reactant.

Why are sharp knives more effective than dull knives? (Hint: Think about the definition of pressure.)

The cutting edge of a knife that has been sharpened has a smaller surface area than a dull knife. Since pressure is force per unit area, a sharp knife will exert a higher pressure with the same amount of force and cut through material more effectively.

Quantitative analysis

The determination of the amount or concentration of a substance in a sample

Guidelines for assigning oxidation numbers to each element in a molecule or ion

The oxidation number of an atom in an elemental substance is zero; The oxidation number of a monatomic ion is equal to the ion's charge; The sum of oxidation numbers for all atoms in a molecule or polyatomic ion equals the charge on the molecule or ion

If the volume of a fixed amount of a gas is tripled at constant temperature, what happens to the pressure?

The pressure decreases by a factor of 3.

How are the molecular mass and the molar mass of a compound similar and how are they different?

The two masses have the same numerical value, but the units are different: The molecular mass is the mass of 1 molecule while the molar mass is the mass of 6.022 × 10^23 molecules.

Explain the difference between extensive properties and intensive properties.

The value of an extensive property depends upon the amount of matter being considered, whereas the value of an intensive property is the same regardless of the amount of matter being considered.

Kinetic molecular theory

Theory based on simple principles and assumptions that effectively explains ideal gas behavior

Law of conservation of matter

There is no detectable change in the tool quantity of matter present when matter converts from one type to another or changes among solid, liquid, or gaseous states

Would you agree to buy 1 trillion (1,000,000,000,000) gold atoms for $5? Explain why or why not. Find the current price of gold at http://money.cnn.com/data/commodities/ (1 troy ounce=31.1 g)

This amount cannot be weighted by ordinary balances and is worthless

A sample of gas isolated from unrefined petroleum contains 90.0% CH4, 8.9% C2H6, and 1.1% C3H8 at a total pressure of 307.2 kPa. What is the partial pressure of each component of this gas? (The percentages given indicate the percent of the total pressure that is due to each component.)

CH4: 276 kPa; C2H6: 27 kPa; C3H8: 3.4 kPa

Identify the postulate of Dalton's theory that is violated by the following observations: 59.95% of one sample of titanium dioxide is titanium; 60.10% of a different sample of titanium dioxide is titanium.

This statement violates Dalton's fourth postulate: In a given compound, the numbers of atoms of each type (and thus also the percentage) always have the same ratio.

What did Thomson's cathode ray tube show?

Thomson's cathode ray tube showed that atoms contain small, negatively charged particles called electrons

Dalton's Law of partial pressures

Total pressure of a mixture of ideal gases is equal to the sum of the partial pressures of the component gases

Effusion

Transfer of gaseous atoms or molecules from a container to a vacuum through very small openings

The average atomic masses of some elements may vary, depending upon the sources of their ores. Naturally occurring boron consists of two isotopes with accurately known masses (10B, 10.0129 amu and 11B, 11.0931 amu). The actual atomic mass of boron can vary from 10.807 to 10.819, depending on whether the mineral source is from Turkey or the United States. Calculate the percent abundances leading to the two values of the average atomic masses of boron from these two countries.

Turkey source: 26.49% (of 10.0129 amu isotope); US source: 25.37% (of 10.0129 amu isotope)

Binary ionic compounds

Typically consist of a metal and a nonmetal

Neutron

Uncharged, subatomic particle located in the nucleus; relatively heavy particles with no charge and a mass of 1.0087 amu

Molarity (M)

Unit of concentration, defined as the number of atoms, molecules, ions, or other entities as the number of atoms in exactly 12 grams of 12C

Atomic mass unit (amu)

Unit of mass equal to 1/12 of the mass of a 12C atom and is equal to 1.6605 * 10^-24 g

Pounds per square inch (psi)

Unit of pressure common in the US

Atmosphere (atm)

Unit of pressure; 1 atm = 101,325 Pa

Bar

Unit of pressure; 1 bar = 100,000 Pa

Torr

Unit of pressure; 1 torr = 1 / 760 atm

Celsius

Unit of temperature; Water freezes at 0°C and boils at 100°C on this scale

Fahrenheit

Unit of temperature; Water freezes at 32°F and boils at 212°F on this scale

Liter

Unit of volume; 1 L = 1,000 cm3

Write a sentence that describes how to determine the number of moles of a compound in a known mass of the compound using its molecular formula.

Use the molecular formula to find the molar mass; to obtain the number of moles, divide the mass of compound by the molar mass of the compound expressed in grams

How are binary acids named?

Using the prefix hydro-, changing the -ide suffix to -ic, and adding "acid;" HCl is hydrochloric acid; The word hydrogen is changed to the prefix hydro; The other nonmetallic element name is modified by adding the suffix -ic; The word "acid" is added as a second word

What is Charles's law

V/T = constant at constant P and n; If the temperature of a gas is increased, a constant pressure may be maintained only if the volume occupied by the gas increases; Will result in greater average distances traveled by the molecules to reach the container walls, as well as increased wall surface area; These conditions will decrease both the frequency of molecule wall collisions and the number of collisions per unit area, the combined effects of balance the effect of increased collision forces due to the greater kinetic energy at the higher temperature

What is Avogradro's law

V/n = constant at constant P and T; At constant pressure and temperature, the frequency and force of molecule wall collisions are constant; Under such conditions, increasing the number of gaseous molecules will require a proportional increase in the container volume in order to yield a decrease in the number of collisions per unit area to compensate for the increased frequency of collisions

Dimensional analysis

Versatile mathematical approach that can be applied to computations ranging from simple unit conversions to more complex, multistep calculations involving several different quantities

Group

Vertical column of the periodic table

Combustion reaction

Vigorous redox reaction producing significant amounts of energy in the form of heat and sometimes, light

Standard molar volume

Volume of 1 mole of gas at STP, approximately 22.4 L for gases behaving ideally

Cubic centimeter

Volume of a cube with an edge length of exactly 1 cm

Avogadro's law

Volume of a gas at constant temperature and pressure is proportional to the number of gas molecules; Under the same conditions of temperature and pressure, equal volumes of all gases contain the same number of molecules

Boyle's law

Volume of a given number of moles of gas held at constant temperature is inversely proportional to the pressure under which it is measured; The volume of a given amount of gas is inversely proportional to its pressure when temperature is held constant

Charle's law

Volume of a given number of moles of gas is directly proportional to its kelvin temperature when the pressure is held constant

What information is needed to calculate the molarity of a sulfuric acid solution?

We need to know the number of moles of sulfuric acid dissolved in the solution and the volume of the solution.

Theories

Well substantiated, comprehensive, testable explanations of particular aspects of nature

Chloride

CL-

Cyanide

CN -

Carbonate

CO3 -2

Calcium

Ca+2

Cobalt (II)

Co+2

Covalent compound

Composed of molecules formed by atoms of two or more different elements

Molecular compound

Composed of molecules formed by atoms of two or more different elements

Mixture

Composed of two or more types of after that can be present in varying amounts and can be separated by physical changes

Ionic compound

Compound composed of cations and anions combined in ratios, yielding an electrically neutral substance

Hydrate

Compound containing one or more water molecules bound within its crystals

Binary compound

Compound containing two different elements

Binary acid

Compound that contains hydrogen and one other element, bonded in way that imparts acidic properties to the compound; Ability to release H+ ions when dissolved in water

Oxyacid

Compound that contains hydrogen, oxygen, and one other element, bonded in a way that imparts acidic properties to the compound

Spatial isomers

Compounds in which the relative orientations of the atoms in space differ

Isomers

Compounds with the same chemical formula but different structures

Mole fraction (X)

Concentration unit defined as the ratio of the molar amount of a mixture component to the total number of moles of all mixture components

Molecule

Consists of two or more atoms joined by chemical bonds

Ideal gas constant (R)

Constant derived from the ideal gas equation R = 0.08206 L atm mol-1 K-1 or 8.314 L kPa mol-1K-1

Outline the steps needed to solve the following problem, then do the calculations. Ether, (C2H5)2O, which was originally used as an anesthetic but has been replaced by safer and more effective medications, is prepared by the reaction of ethanol with sulfuric acid. 2C2H5OH + H2SO4 ⟶ (C2H5)2O + H2SO4·H2O What is the percent yield of ether if 1.17 L (d = 0.7134 g/mL) is isolated from the reaction of 1.500 L of C2H5OH (d = 0.7894 g/mL)?

Convert mass of ethanol to moles of ethanol; relate the moles of ethanol to the moles of ether produced using the stoichiometry of the balanced equation. Convert moles of ether to grams; divide the actual grams of ether (determined through the density) by the theoretical mass to determine the percent yield; 87.6%

Chromium (III)

Cr+3

Cesium

Cs+

Copper (II)

Cu+2

The existence of isotopes violates one of the original ideas of Dalton's atomic theory. Which one?

Dalton originally thought that all atoms of a particular element had identical properties, including mass. Thus, the concept of isotopes, in which an element has different masses, was a violation of the original idea. To account for the existence of isotopes, the second postulate of his atomic theory was modified to state that atoms of the same element must have identical chemical properties.

How to find density

Density = mass / volume

Extensive property

Depends on the amount of matter present

Dissolved

Describes the process by which solute components are dispersed in a solvent

Buret

Device used for the precise delivery of variable liquid volumes, such as in a titration analysis

Barometer

Device used to measure atmospheric pressure

Manometer

Device used to measure the pressure of a gas trapped in a container

Intensive property

Does not depend on the amount of matter present

Metal

Element that is shiny, malleable, good conductor of heat and electricity; tend to lose the number of electrons that would leave them with the same number of electrons as in the preceding noble gas in the periodic table

Nonmetals

Elements that conduct poorly

Metals

Elements that conduct well

Metalloids

Elements that have intermediate conductivities

Fundamental unit of charge

Equals the magnitude of the charge of an electron

Uncertainty

Estimate of amount by which measurement differs from true value

Homogeneous mixture

Exhibits a uniform composition and appears visually the same throughout

Avogadro's number (NA)

Experimentally determined value of the number of entities comprising 1 mole of substance, equal to 6.022 x 1023 mol-1

Iron (II)

Fe+2

Weight

Force that gravity exerts on an object which is proportional to the mass of the object

Which contains the greatest mass of oxygen: 0.75 mol of ethanol (C2H5OH), 0.60 mol of formic acid (HCO2H), or 1.0 mol of water (H2O)? Explain why.

Formic acid. Its formula has twice as many oxygen atoms as the other two compounds (one each). Therefore, 0.60 mol of formic acid would be equivalent to 1.20 mol of a compound containing a single oxygen atom

Molecular formula

Formula indicating the composition of a molecule of a compound and giving the actual number of atoms of each element in a molecule of the compound

Empirical formula

Formula showing the composition of a compound given as the simplest whole number ratio of atoms

Graphs showing the behavior of several different gases follow. Which of these gases exhibit behavior significantly different from that expected for ideal gases?

Gases C, E, and F

What are the five postulates of the Kinetic molecular theory

Gases are composed of molecules that are in continuous motion, travelling in straight lines and changing direction only when they collide with other molecules or with the walls of a container; The molecules composing the gas are negligibly small compared to the distances between them; The pressure exerted by a gas in a container results from collisions between the gas molecules and the container walls; Gas molecules exert no attractive or repulsive forces on each other or the container walls; therefore, their collisions are elastic (do not involve a loss of energy); The average kinetic energy of the gas molecules is proportional to the kelvin temperature of the gas

Prepare a list of the principal chemicals consumed and produced during the operation of an automobile.

Gasoline (a mixture of compounds), oxygen, and to a lesser extent, nitrogen are consumed. Carbon dioxide and water are the principal products. Carbon monoxide and nitrogen oxides are produced in lesser amounts.

Combustion analysis

Gravimetric technique used to determine the elemental composition of a compound via the collection and weighing of its gaseous combustion products

Hydride

H-

The military has experimented with lasers that produce very intense light when fluorine combines explosively with hydrogen. What is the balanced equation for this reaction?

H2(g)+F2(g) ⟶ 2HF(g)

The distribution of molecular velocities in a sample of helium is shown in Figure 9.34. If the sample is cooled, will the distribution of velocities look more like that of H2 or of H2O? Explain your answer.

H2O. Cooling slows the velocities of the He atoms, causing them to behave as though they were heavier.

Dihydrogen Phosphate

H2PO4 -

Hydronium

H3O+

Hydrogen carbonate

HCO3 -

Hydrogen phosphate

HPO4 -2

Hydrogen sulfate

HSO4 -

Mercury (II)

Hg+2

Mercury (I)

Hg2 +2

Period

Horizontal row of the periodic table

Series

Horizontal row of the periodic table

Temperature

Hotness or coldness of a substance

Accuracy

How closely a measurement aligns with a correct value

Precision

How closely a measurement matches the same measurement when repeated

Oxidation numbers for common nonmetals are usually assigned as follows

Hydrogen: +1 when combined with nonmetals, -1 when combined with metals; Oxygen: -2 in most compounds, sometimes -1 (so called peroxides, O22-), very rarely -½ (so-called superoxides, O2-), positive values when combined with F (values vary); Halogens: -1 for F always, -1 for other halogens except when combined with oxygen or other halogens (positive oxidation numbers in these cases, varying values)

Iodide

I-

Spectator ion

Ion that does not undergo a chemical or physical change during a reaction, but its presence is required to maintain charge neutrality

Salt

Ionic compound that can be formed by the reaction of an acid with a base that contains a cation and an anion other than hydroxide or oxide

Using the periodic table, predict whether the following chlorides are ionic or covalent: KCl, NCl3, ICl, MgCl2, PCl5, and CCl4.

Ionic: KCl, MgCl2; Covalent: NCl3, ICl, PCl5, CCl4

Silver can be separated from gold because silver dissolves in nitric acid while gold does not. Is the dissolution of silver in nitric acid an acid-base reaction or an oxidation-reduction reaction? Explain your answer.

It is an oxidation-reduction reaction because the oxidation state of the silver changes during the reaction

End point

Measured volume of titrant solution that yields the change in sample solution appearance or other property expected for stoichiometric amount of titrant for the sample's analyte according to the titration reaction

Magnesium

Mg+2

Determine the empirical and molecular formula for chrysotile asbestos. Chrysotile has the following percent composition: 28.03% Mg, 21.60% Si, 1.16% H, and 49.21% O. The molar mass for chrysotile is 520.8 g/mol.

Mg3Si2H3O8 (empirical formula), Mg6Si4H6O16 (molecular formula)

What did Milikan discover?

Millikan discovered that there is a fundamental electric charge—the charge of an electron

What did Rutherford's experiment show?

Millikan discovered that there is a fundamental electric charge—the charge of an electron

Heterogeneous mixture

Mixture with a composition that varies from point to point

Manganese (II)

Mn+2

Permanganate

MnO4 -

Van der Waals equation

Modified version of the ideal gas equation containing additional terms to account for non ideal gas behavior

How do molecules of elements and molecules of compounds differ? In what ways are they similar?

Molecules of elements contain only one type of atom; molecules of compounds contain two or more types of atoms. They are similar in that both are comprised of two or more atoms chemically bonded together.

What is Dalton's law

Molecules of one gas in a mixture bombard the container walls with the same frequency whether other gases are present or not, and the total pressure of a gas mixture equals the sum of the (partial) pressures of the individual gases

Diffusion

Movement of an atom or molecule from a region of relatively high concentration to one of relatively low concentration

Ammonium

NH4 +

Nitrite

NO2 -

Nitrate

NO3 -

Sodium

Na+

Uranium can be isolated from its ores by dissolving it as UO2(NO3)2, then separating it as solid UO2(C2O4)·3H2O. Addition of 0.4031 g of sodium oxalate, Na2C2O4, to a solution containing 1.481 g of uranyl nitrate, UO2(NO3)2, yields 1.073 g of solid UO2(C2O4)·3H2O. Na2C2O4 + UO2(NO3)2 + 3H2O ⟶ UO2(C2O4)·3H2O + 2NaNO3 Determine the limiting reactant and the percent yield of this reaction.

Na2C2O4 is the limiting reactant. percent yield = 86.6%

How are compounds formatted?

Name of the metal is written first, followed by the name of the nonmetal with its ending changed to -ide; If the metal can form ions with different charges, a Roman numeral in parentheses follows the name of the metal to specify its charge

Anion

Negatively charged atom or molecule; Contains more electrons than protons

Electron

Negatively charged, subatomic particle of relatively low mass located outside the nucleus; a charge of 1− and a mass of 0.00055 amu

Nickel (II)

Ni+2

Exact number

Number derived by counting or by definition

Atomic number (Z)

Number of protons in the nucleus of an atom; property that defines an atom's elemental identity

Coefficient

Number placed in front of symbols or formulas in a chemical equation to indicate their relative amount

Oxide

O -2

Peroxide

O2 -2

Hydroxide

OH -

Soluble

Of relatively high solubility; Dissolving to a relatively large extent

Insoluble

Of relatively low solubility; Dissolving only to a slight extent

Structural isomer

One of two substances that have the same molecular formula but different physical and chemical properties because their atoms are bonded differently

Chemical symbol

One-, two-, or three-letter abbreviation used to represent an element or its atoms

How many molecules of the sweetener saccharin can be prepared from 30 C atoms, 25 H atoms, 12 O atoms, 8 S atoms, and 14 N atoms?

Only four molecules can be made.

How are oxyacids named?

Oxyacids are named by changing the ending of the anion (-ate to -ic and -ite to -ous), and adding "acid;" H2CO3 is carbonic acid; Omit hydrogen; Start with the root name of the anion; Replace -ate with -ic, ir -ite with -ous

1 atom

mono-

9 atoms

nona-

8 atoms

octa-

Law of multiple proportions

When two elements react to form more than one compound, a fixed mass of one element will react with masses of the other element in a ratio of small whole numbers

How would the use of a volatile liquid affect the measurement of a gas using open-ended manometers vs. closed-end manometers?

With a closed-end manometer, no change would be observed, since the vaporized liquid would contribute equal, opposing pressures in both arms of the manometer tube. However, with an open-ended manometer, a higher pressure reading of the gas would be obtained than expected, since Pgas =Patm +Pvol liquid.

A sample of a compound of xenon and fluorine was confined in a bulb with a pressure of 18 torr. Hydrogen was added to the bulb until the pressure was 72 torr. Passage of an electric spark through the mixture produced Xe and HF. After the HF was removed by reaction with solid KOH, the final pressure of xenon and unreacted hydrogen in the bulb was 36 torr. What is the empirical formula of the xenon fluoride in the original sample?

XeF2

Most mixtures of hydrogen gas with oxygen gas are explosive. However, a mixture that contains less than 3.0 % O2 is not. If enough O2 is added to a cylinder of H2 at 33.2 atm to bring the total pressure to 34.5 atm, is the mixture explosive?

Yes

As an instructor is preparing for an experiment, he requires 225 g phosphoric acid. The only container readily available is a 150-mL Erlenmeyer flask. Is it large enough to contain the acid, whose density is 1.83 g/mL?

Yes, the acid's volume is 123 mL

Can the speed of a given molecule in a gas double at constant temperature? Explain your answer.

Yes. At any given instant, there are a range of values of molecular speeds in a sample of gas. Any single molecule can speed up or slow down as it collides with other molecules. The average velocity of all the molecules is constant at constant temperature

2 atoms

di-

5 atoms

penta-

Toluene, C6H5CH3, is oxidized by air under carefully controlled conditions to benzoic acid, C6H5CO2H, which is used to prepare the food preservative sodium benzoate, C6H5CO2Na. What is the percent yield of a reaction that converts 1.000 kg of toluene to 1.21 kg of benzoic acid? 2C6H5CH3+3O2 ⟶ 2C6H5CO2H+2H2O

percent yield=91.3%

The volume of a sample of oxygen gas changed from 10 mL to 11 mL as the temperature changed. Is this a chemical or physical change?

physical

How do you calculate parts per billion

ppb = (mass solute / mass solution) * 109 ppb

How do you calculate parts per million

ppm = (mass solute / mass solution) * 106 ppm

3 atoms

tri-

Is a 197-lb weight lifter light enough to compete in a class limited to those weighing 90 kg or less?

yes; weight = 89.4 kg

Determine the number of atoms and the mass of zirconium, silicon, and oxygen found in 0.3384 mol of zircon, ZrSiO4, a semiprecious stone.

zirconium: 2.038 × 10^23 atoms; 30.87 g; silicon: 2.038 × 10^23 atoms; 9.504 g; oxygen: 8.151 × 10^23 atoms; 21.66 g

Convert the temperature of the coldest area in a freezer, −10 °F, to degrees Celsius and kelvin.

−23 °C, 250 K

Convert the boiling temperature of liquid ammonia, −28.1 °F, into degrees Celsius and kelvin.

−33.4 °C, 239.8 K

Main-group element

Another term for representative element

Period

Another term for series

Answer the following questions: (a) If XX behaved as an ideal gas, what would its graph of Z vs. P look like? (b) For most of this chapter, we performed calculations treating gases as ideal. Was this justified? (c) What is the effect of the volume of gas molecules on Z? Under what conditions is this effect small? When is it large? Explain using an appropriate diagram. (d) What is the effect of intermolecular attractions on the value of Z? Under what conditions is this effect small? When is it large? Explain using an appropriate diagram. (e) In general, under what temperature conditions would you expect Z to have the largest deviations from the Z for an ideal gas?

(a) A straight horizontal line at 1.0; (b) When real gases are at low pressures and high temperatures, they behave close enough to ideal gases that they are approximated as such; however, in some cases, we see that at a high pressure and temperature, the ideal gas approximation breaks down and is significantly different from the pressure calculated by the ideal gas equation. (c) The greater the compressibility, the more the volume matters. At low pressures, the correction factor for intermolecular attractions is more significant, and the effect of the volume of the gas molecules on Z would be a small lowering compressibility. At higher pressures, the effect of the volume of the gas molecules themselves on Z would increase compressibility (see Figure9.35). (d) Once again, at low pressures, the effect of intermolecular attractions on Z would be more important than the correction factor for the volume of the gas molecules themselves, though perhaps still small. At higher pressures and low temperatures, the effect of intermolecular attractions would be larger. See Figure9.35. (e) Low temperatures

Colorful fireworks often involve the decomposition of barium nitrate and potassium chlorate and the reaction of the metals magnesium, aluminum, and iron with oxygen. (a) Write the formulas of barium nitrate and potassium chlorate. (b) The decomposition of solid potassium chlorate leads to the formation of solid potassium chloride and diatomic oxygen gas. Write an equation for the reaction. (c) The decomposition of solid barium nitrate leads to the formation of solid barium oxide, diatomic nitrogen gas, and diatomic oxygen gas. Write an equation for the reaction. (d) Write separate equations for the reactions of the solid metals magnesium, aluminum, and iron with diatomic oxygen gas to yield the corresponding metal oxides. (Assume the iron oxide contains Fe3+ ions.)

(a) Ba(NO3)2, KClO3; (b) 2KClO3(s) ⟶ 2KCl(s)+3O2(g); (c) 2Ba(NO3)2(s) ⟶ 2BaO(s)+2N2(g)+5O2(g); (d) 2Mg(s)+O2(g) ⟶ 2MgO(s); 4Al(s)+3O2(g) ⟶ 2Al2O3(s); 4Fe(s)+3O2(g) ⟶ 2Fe2O3(s)

Lime, CaO, is produced by heating calcium carbonate, CaCO3; carbon dioxide is the other product. (a) Outline the steps necessary to answer the following question: What volume of carbon dioxide at 875 K and 0.966 atm is produced by the decomposition of 1 ton (1.000 × 10^3 kg) of calcium carbonate? (b) Answer the question.

(a) Balance the equation. Determine the grams of CO2 produced and the number of moles. From the ideal gas law, determine the volume of gas. (b) 7.43 × 105 L

Write the empirical formulas for the following compounds:

(a) C4H5N2O; (b) C12H22O11; (c) HO; (d) CH2O; (e) C3H4O3

Determine the empirical formulas for compounds with the following percent compositions: (a) 15.8% carbon and 84.2% sulfur (b) 40.0% carbon, 6.7% hydrogen, and 53.3% oxygen

(a) CS2; (b) CH2O

Complete and balance the equations of the following reactions, each of which could be used to remove hydrogen sulfide from natural gas: (a) Ca(OH)2(s)+H2S(g) ⟶ (b) Na2CO3(aq)+H2S(g) ⟶

(a) Ca(OH)2(s)+H2S(g) ⟶ CaS(s)+2H2O(l); (b) Na2CO3(aq)+H2S(g) ⟶ Na2S(aq)+CO2(g)+H2O(l)

Write a balanced molecular equation describing each of the following chemical reactions. (a) Solid calcium carbonate is heated and decomposes to solid calcium oxide and carbon dioxide gas. (b) Gaseous butane, C4H10, reacts with diatomic oxygen gas to yield gaseous carbon dioxide and water vapor. (c) Aqueous solutions of magnesium chloride and sodium hydroxide react to produce solid magnesium hydroxide and aqueous sodium chloride. (d) Water vapor reacts with sodium metal to produce solid sodium hydroxide and hydrogen gas.

(a) CaCO3(s) ⟶ CaO(s)+CO2(g); (b) 2C4H10(g)+13O2(g) ⟶ 8CO2(g)+10H2O(g); (c) MgC12(aq)+2NaOH(aq) ⟶ Mg(OH)2(s)+2NaCl(aq); (d) 2H2O(g)+2Na(s) ⟶ 2NaOH(s)+H2(g)

For each of the following pairs of ions, write the formula of the compound they will form: (a) Ca2+, S2− (b) NH4+ , SO42− (c) Al3+, Br− (d) Na+, HPO42− (e) Mg2+, PO43−

(a) CaS; (b) (NH4)2SO4; (c) AlBr3; (d) Na2HPO4; (e) Mg3 (PO4)2

Write the formulas of the following compounds: (a) chlorine dioxide (b) dinitrogen tetraoxide (c) potassium phosphide (d) silver(I) sulfide (e) aluminum fluoride trihydrate (f) silicon dioxide

(a) ClO2; (b) N2O4; (c) K3P; (d) Ag2S; (e) AIF3·3H2O; (f) SiO2

The chlorofluorocarbon CCl2F2 can be recycled into a different compound by reaction with hydrogen to produce CH2F2(g), a compound useful in chemical manufacturing: CCl2F2(g)+4H2(g) ⟶ CH2F2(g)+2HCl(g) (a) Outline the steps necessary to answer the following question: What volume of hydrogen at 225 atm and 35.5 °C would be required to react with 1 ton (1.000 × 10^3 kg) of CCl2F2? (b) Answer the question.

(a) Determine the molar mass of CCl2F2. From the balanced equation, calculate the moles of H2 needed for the complete reaction. From the ideal gas law, convert moles of H2 into volume. (b) 3.72 × 10^3 L

Consider this question: What is the molarity of KMnO4 in a solution of 0.0908 g of KMnO4 in 0.500 L of solution? (a) Outline the steps necessary to answer the question. (b) Answer the question

(a) Determine the molar mass of KMnO4; determine the number of moles of KMnO4 in the solution; from the number of moles and the volume of solution, determine the molarity; (b) 1.15 × 10−3 M

Joseph Priestley first prepared pure oxygen by heating mercuric oxide, HgO: 2HgO(s) ⟶ 2Hg(l)+O2(g) (a) Outline the steps necessary to answer the following question: What volume of O2 at 23 °C and 0.975 atm is produced by the decomposition of 5.36 g of HgO? (b) Answer the question.

(a) Determine the moles of HgO that decompose; using the chemical equation, determine the moles of O2 produced by decomposition of this amount of HgO; and determine the volume of O2 from the moles of O2, temperature, and pressure. (b) 0.308

Determine the oxidation states of the elements in the compounds listed. None of the oxygen-containing compounds are peroxides or superoxides. (a) H3PO4 (b) Al(OH)3 (c) SeO2 (d) KNO2 (e) In2S3 (f) P4O6

(a) H +1, P +5, O −2; (b) Al +3, H +1, O −2; (c) Se +4, O −2; (d) K +1, N +3, O −2; (e) In +3, S −2; (f) P +3, O −2

Balance the following equations: (a) PCl5(s)+H2O(l) ⟶ POCl3(l)+HCl(aq) (b) Cu(s)+HNO3(aq) ⟶ Cu(NO3)2(aq)+H2O(l)+NO(g) (c) H2(g)+I2(s) ⟶ HI(s) (d) Fe(s)+O2(g) ⟶ Fe2O3(s) (e) Na(s)+H2O(l) ⟶ NaOH(aq)+H2(g) (f) (NH4)2Cr2O7(s) ⟶ Cr2O3(s)+N2(g)+H2O(g) (g) P4(s)+Cl2(g) ⟶ PCl3(l) (h) PtCl4(s) ⟶ Pt(s)+Cl2(g)

(a) PCl5(s)+H2O(l) ⟶ POCl3(l)+2HCl(aq); (b) 3Cu(s)+8HNO3(aq) ⟶ 3Cu(NO3)2(aq)+4H2O(l)+2NO(g); (c) H2(g)+I2(s) ⟶ 2HI(s); (d) 4Fe(s)+3O2(g) ⟶ 2Fe2O3(s); (e) 2Na(s)+2H2O(l) ⟶ 2NaOH(aq)+H2(g); (f) (NH4)2Cr2O7(s) ⟶ Cr2O3(s)+N2(g)+4H2O(g); (g) P4(s)+6Cl2(g) ⟶ 4PCl3(l); (h) PtCl4(s) ⟶ Pt(s)+2Cl2(g)

Write the formulas of the following compounds: (a) rubidium bromide (b) magnesium selenide (c) sodium oxide (d) calcium chloride (e) hydrogen fluoride (f) gallium phosphide (g) aluminum bromide (h) ammonium sulfate

(a) RbBr; (b) MgSe; (c) Na2O; (d) CaCl2; (e) HF; (f) GaP; (g) AlBr3; (h) (NH4)2SO4

Predict and test the behavior of α particles fired at a Rutherford atom model. (a) Predict the paths taken by α particles that are fired at atoms with a Rutherford atom model structure. Explain why you expect the α particles to take these paths. (b) If α particles of higher energy than those in (a) are fired at Rutherford atoms, predict how their paths will differ from the lower-energy α particle paths. Explain your reasoning. (c) Predict how the paths taken by the α particles will differ if they are fired at Rutherford atoms of elements other than gold. What factor do you expect to cause this difference in paths, and why?

(a) The Rutherford atom has a small, positively charged nucleus, so most α particles will pass through empty space far from the nucleus and be undeflected. Those α particles that pass near the nucleus will be deflected from their paths due to positive-positive repulsion. The more directly toward the nucleus the α particles are headed, the larger the deflection angle will be. (b) Higher-energy α particles that pass near the nucleus will still undergo deflection, but the faster they travel, the less the expected angle of deflection. (c) If the nucleus is smaller, the positive charge is smaller and the expected deflections are smaller—both in terms of how closely the α particles pass by the nucleus undeflected and the angle of deflection. If the nucleus is larger, the positive charge is larger and the expected deflections are larger—more α particles will be deflected, and the deflection angles will be larger.

Consider this question: What mass of a concentrated solution of nitric acid (68.0% HNO3 by mass) is needed to prepare 400.0 g of a 10.0% solution of HNO3 by mass? (a) Outline the steps necessary to answer the question. (b) Answer the question.

(a) The dilution equation can be used, appropriately modified to accommodate mass-based concentration units: %mass1 ×mass1 =%mass2 ×mass2. This equation can be rearranged to isolate mass1 and the given quantities substituted into this equation. (b) 58.8 g

A 1-L sample of CO initially at STP is heated to 546 K, and its volume is increased to 2 L. (a) What effect do these changes have on the number of collisions of the molecules of the gas per unit area of the container wall? (b) What is the effect on the average kinetic energy of the molecules? (c) What is the effect on the root mean square speed of the molecules?

(a) The number of collisions per unit area of the container wall is constant. (b) The average kinetic energy doubles. (c) The root mean square speed increases to 2 times its initial value; urms is proportional to √(KEavg)

An alternate way to state Avogadro's law is "All other things being equal, the number of molecules in a gas is directly proportional to the volume of the gas." (a) What is the meaning of the term "directly proportional?" (b) What are the "other things" that must be equal?

(a) The number of particles in the gas increases as the volume increases. (b) temperature, pressure

Classify the following as acid-base reactions or oxidation-reduction reactions: (a) Na2S(aq)+2HCl(aq) ⟶ 2NaCl(aq)+H2S(g) (b) 2Na(s)+2HCl(aq) ⟶ 2NaCl(aq)+H2(g) (c) Mg(s)+Cl2(g) ⟶ MgCl2(s) (d) MgO(s)+2HCl(aq) ⟶ MgCl2(aq)+H2O(l) (e) K3P(s)+2O2(g) ⟶ K3PO4(s) (f) 3KOH(aq)+H3PO4(aq) ⟶ K3PO4(aq)+3H2O(l)

(a) acid-base; (b) oxidation-reduction: Na is oxidized, H+ is reduced; (c) oxidation-reduction: Mg is oxidized, Cl2 is reduced; (d) acid-base; (e) oxidation-reduction: P3− is oxidized, O2 is reduced; (f) acid-base

Give the name of the prefix and the quantity indicated by the following symbols that are used with SI base units. (a) c (b) d (c) G (d) k (e) m (f) n (g) p (h) T

(a) centi-, × 10^−2; (b) deci-, × 10^−1; (c) Giga-, × 10^9; (d) kilo-, × 10^3; (e) milli-, × 10^−3; (f) nano-, × 10^−9; (g) pico-, × 10^−12; (h) tera-, × 10^12

Name the following compounds: (a) CsCl (b) BaO (c) K2S (d) BeCl2 (e) HBr (f) AlF3

(a) cesium chloride; (b) barium oxide; (c) potassium sulfide; (d) beryllium chloride; (e) hydrogen bromide; (f) aluminum fluoride

Each of the following compounds contains a metal that can exhibit more than one ionic charge. Name these compounds: (a) Cr2O3 (b) FeCl2 (c) CrO3 (d) TiCl4 (e) CoCl2·6H2O (f) MoS2

(a) chromium(III) oxide; (b) iron(II) chloride; (c) chromium(VI) oxide; (d) titanium(IV) chloride; (e) cobalt(II) chloride hexahydrate; (f) molybdenum(IV) sulfid

For each of the following compounds, state whether it is ionic or covalent. If it is ionic, write the symbols for the ions involved: (a) NF3 (b) BaO (c) (NH4)2CO3 (d) Sr(H2PO4)2 (e) IBr (f) Na2O

(a) covalent; (b) ionic, Ba2+, O2−; (c) ionic, NH4+ , CO32−; (d) ionic, Sr2+, H2PO4−; (e) covalent; (f) ionic, Na+, O2

Consider this question: What is the mass of the solute in 0.500 L of 0.30 Mglucose, C6H12O6, used for intravenous injection? (a) Outline the steps necessary to answer the question. (b) Answer the question.

(a) determine the number of moles of glucose in 0.500 L of solution; determine the molar mass of glucose; determine the mass of glucose from the number of moles and its molar mass; (b) 27 g

If 0.1718 L of a 0.3556-MC3H7OH solution is diluted to a concentration of 0.1222 M, what is the volume of the resulting solution?

0.5000 L

Oxidation stage

Another term for oxidation number

Answer the following questions: (a) Is the pressure of the gas in the hot-air balloon shown at the opening of this chapter greater than, less than, or equal to that of the atmosphere outside the balloon? (b) Is the density of the gas in the hot-air balloon shown at the opening of this chapter greater than, less than, or equal to that of the atmosphere outside the balloon? (c) At a pressure of 1 atm and a temperature of 20 °C, dry air has a density of 1.2256 g/L. What is the (average) molar mass of dry air? (d) The average temperature of the gas in a hot-air balloon is 1.30 × 102 °F. Calculate its density, assuming the molar mass equals that of dry air. (e) The lifting capacity of a hot-air balloon is equal to the difference in the mass of the cool air displaced by the balloon and the mass of the gas in the balloon. What is the difference in the mass of 1.00 L of the cool air in part (c) and the hot air in part (d)? (f) An average balloon has a diameter of 60 feet and a volume of 1.1 × 105 ft3. What is the lifting power of such a balloon? If the weight of the balloon and its rigging is 500 pounds, what is its capacity for carrying passengers and cargo? (g) A balloon carries 40.0 gallons of liquid propane (density 0.5005 g/L). What volume of CO2 and H2O gas is produced by the combustion of this propane? (h) A balloon flight can last about 90 minutes. If all of the fuel is burned during this time, what is the approximate rate of heat loss (in kJ/min) from the hot air in the bag during the flight?

(a) equal; (b) less than; (c) 29.48 g mol^−1; (d) 1.0966 g L^−1; (e) 0.129 g/L; (f) 4.01 × 10^5 g; net lifting capacity = 384 lb; (g) 270 L; (h) 39.1 kJ min^−1

Indicate whether each of the following can be determined exactly or must be measured with some degree of uncertainty: (a) the number of seconds in an hour (b) the number of pages in this book (c) the number of grams in your weight (d) the number of grams in 3 kilograms (e) the volume of water you drink in one day (f) the distance from San Francisco to Kansas City

(a) exact; (b) exact; (c) uncertain; (d) exact; (e) uncertain; (f) uncertain

Indicate the SI base units or derived units that are appropriate for the following measurements: (a) the mass of the moon (b) the distance from Dallas to Oklahoma City (c) the speed of sound (d) the density of air (e) the temperature at which alcohol boils (f) the area of the state of Delaware (g) the volume of a flu shot or a measles vaccination

(a) kilograms; (b) meters; (c) kilometers/second; (d) kilograms/cubic meter; (e) kelvin; (f) square meters; (g) cubic meters

Use the periodic table to give the name and symbol for each of the following elements: (a) the noble gas in the same period as germanium (b) the alkaline earth metal in the same period as selenium (c) the halogen in the same period as lithium (d) the chalcogen in the same period as cadmium

(a) krypton, Kr; (b) calcium, Ca; (c) fluorine, F; (d) tellurium, Te

3. Identify each of the following statements as being most similar to a hypothesis, a law, or a theory. Explain your reasoning. (a) The pressure of a sample of gas is directly proportional to the temperature of the gas. (b) Matter consists of tiny particles that can combine in specific ratios to form substances with specific properties. (c) At a higher temperature, solids (such as salt or sugar) will dissolve better in water.

(a) law (states a consistently observed phenomenon, can be used for prediction); (b) theory (a widely accepted explanation of the behavior of matter); (c) hypothesis (a tentative explanation, can be investigated by experimentation)

What are the IUPAC names of the following compounds? (a) manganese dioxide (b) mercurous chloride (Hg2Cl2) (c) ferric nitrate [Fe(NO3)3] (d) titanium tetrachloride (e) cupric bromide (CuBr2)

(a) manganese(IV) oxide; (b) mercury(I) chloride; (c) iron(III) nitrate; (d) titanium(IV) chloride; (e) copper(II) bromide

Using the periodic table, classify each of the following elements as a metal or a nonmetal, and then further classify each as a main-group (representative) element, transition metal, or inner transition metal: (a) uranium (b) bromine (c) strontium (d) neon (e) gold (f) americium (g) rhodium (h) sulfur (i) carbon (j) potassium

(a) metal, inner transition metal; (b) nonmetal, representative element; (c) metal, representative element; (d) nonmetal, representative element; (e) metal, transition metal; (f) metal, inner transition metal; (g) metal, transition metal; (h) nonmetal, representative element; (i) nonmetal, representative element; (j) metal, representative element

Write the molecular and empirical formulas of the following compounds:

(a) molecular CO2, empirical CO2; (b) molecular C2H2, empirical CH; (c) molecular C2H4, empirical CH2; (d) molecular H2SO4, empirical H2SO4

Indicate what type, or types, of reaction each of the following represents: (a) Ca(s)+Br2(l) ⟶ CaBr2(s) (b) Ca(OH)2(aq)+2HBr(aq) ⟶ CaBr2(aq)+2H2O(l) (c) C6H12(l)+9O2(g) ⟶ 6CO2(g)+6H2O(g)

(a) oxidation-reduction (addition); (b) acid-base (neutralization); (c) oxidation-reduction (combustion)

Classify each of the following changes as physical or chemical: (a) condensation of steam (b) burning of gasoline (c) souring of milk (d) dissolving of sugar in water (e) melting of gold

(a) physical; (b) chemical; (c) chemical; (d) physical; (e) physical

Write balanced chemical equations for the reactions used to prepare each of the following compounds from the given starting material(s). In some cases, additional reactants may be required. (a) solid ammonium nitrate from gaseous molecular nitrogen via a two-step process (first reduce the nitrogen to ammonia, then neutralize the ammonia with an appropriate acid) (b) gaseous hydrogen bromide from liquid molecular bromine via a one-step redox reaction (c) gaseous H2S from solid Zn and S via a two-step process (first a redox reaction between the starting materials, then reaction of the product with a strong acid)

(a) step 1: N2(g)+3H2(g) ⟶ 2NH3(g), step 2: NH3(g)+HNO3(aq) ⟶ NH4NO3(aq) ⟶ NH4NO3(s)(after drying); (b) H2(g)+Br2(l) ⟶ 2HBr(g); (c) Zn(s)+S(s) ⟶ ZnS(s) and ZnS(s)+2HCl(aq) ⟶ ZnCl2(aq)+H2S(g)

Identify each of the underlined items as a part of either the macroscopic domain, the microscopic domain, or the symbolic domain of chemistry. For those in the symbolic domain, indicate whether they are symbols for a macroscopic or a microscopic feature. (a) A certain molecule contains one H atom and one Cl atom. (b) Copper wire has a density of about 8 g/cm^3. (c) The bottle contains 15 grams of Ni powder. (d) A sulfur molecule is composed of eight sulfur atoms.

(a) symbolic, microscopic; (b) macroscopic; (c) symbolic, macroscopic; (d) microscopic

How many significant figures are contained in each of the following measurements? (a) 53 cm (b) 2.05 × 10^8 m (c) 86,002 J (d) 9.740 × 10^4 m/s (e) 10.0613 m^3 (f) 0.17 g/mL (g) 0.88400 s

(a) two; (b) three; (c) five; (d) four; (e) six; (f) two; (g) five

Calculate the density of aluminum if 27.6 cm^3 has a mass of 74.6 g.

.2.70 g/cm3

A sample of solid calcium hydroxide, Ca(OH)2, is allowed to stand in water until a saturated solution is formed. A titration of 75.00 mL of this solution with 5.00 × 10−2 MHCl requires 36.6 mL of the acid to reach the end point. Ca(OH)2(aq)+2HCl(aq) ⟶ CaCl2(aq)+2H2O(l): What is the molarity?

0.0122M

There is about 1.0 g of calcium, as Ca2+, in 1.0 L of milk. What is the molarity of Ca2+ in milk?

0.025M

D5W is a solution used as an intravenous fluid. It is a 5.0% by mass solution of dextrose (C6H12O6) in water. If the density of D5W is 1.029 g/mL, calculate the molarity of dextrose in the solution.

0.29 mol

A chemist's 50-Trillion Angstrom Run (see Exercise 1.78) would be an archeologist's 10,900 cubit run. How long is one cubit in meters and in feet? (1 Å = 1 × 10^−8 cm)

0.46 m; 1.5 ft/cubit

A long ton is defined as exactly 2240 lb. What is this mass in kilograms?

1.0160 × 103 kg

A compact car gets 37.5 miles per gallon on the highway. If gasoline contains 84.2% carbon by mass and has a density of 0.8205 g/mL, determine the mass of carbon dioxide produced during a 500-mile trip (3.785 liters per gallon).

1.28 × 105 g CO2

Calculate the relative rate of diffusion of 1H2 (molar mass 2.0 g/mol) compared with 2H2 (molar mass 4.0 g/ mol) and the relative rate of diffusion of O2 (molar mass 32 g/mol) compared with O3 (molar mass 48 g/mol).

1.4; 1.2

What volume of a 0.20-MK2SO4 solution contains 57 g of K2SO4?

1.6 L

The hardness of water (hardness count) is usually expressed in parts per million (by mass) of CaCO3, which is equivalent to milligrams of CaCO3 per liter of water. What is the molar concentration of Ca2+ions in a water sample with a hardness count of 175 mg CaCO3/L?

1.75 × 10−3 M

What volume of a 0.33-MC12H22O11 solution can be diluted to prepare 25 mL of a solution with a concentration of 0.025M

1.9 mL

Milliliter

1/1,000 of of a liter Equal to 1 cm3

What volume of oxygen at 423.0 K and a pressure of 127.4 kPa is produced by the decomposition of 129.7 g of BaO2 to BaO and O2?

10.57 L O2

centi (c)

10^-2 (0.01)

milli (m)

10^-3 (0.001)

micro (μ)

10^-6 (0.000001)

nano (n)

10^-9 (0.000000001)

kilo (k)

10^3 (1,000)

giga (G)

10^9 (1,000,000,000)

A 2.00-L bottle of a solution of concentrated HCl was purchased for the general chemistry laboratory. The solution contained 868.8 g of HCl. What is the molarity of the solution?

11.9M

The weather in Europe was unusually warm during the summer of 1995. The TV news reported temperatures as high as 45 °C. What was the temperature on the Fahrenheit scale?

113 °F

What mass of solid NaOH (97.0% NaOH by mass) is required to prepare 1.00 L of a 10.0% solution of NaOH by mass? The density of the 10.0% solution is 1.109 g/mL.

114 g

A 2.50-L volume of hydrogen measured at -196 °C is warmed to 100 °C. Calculate the volume of the gas at the higher temperature, assuming no change in pressure.

12.1 L

What volume of a 0.3300-Msolution of sodium hydroxide would be required to titrate 15.00 mL of 0.1500 M oxalic acid? C2O4H2(aq)+2NaOH(aq) ⟶ Na2C2O4(aq)+2H2O(l)

13.64 mL

A 36.0-L cylinder of a gas used for calibration of blood gas analyzers in medical laboratories contains 350 g CO2, 805 g O2, and 4,880 g N2. At 25 degrees C, what is the pressure in the cylinder in atmospheres?

141 atm, 107,000 torr, 14,300 kPa

What volume of a 0.2089MKI solution contains enough KI to react exactly with the Cu(NO3)2 in 43.88 mL of a 0.3842Msolution of Cu(NO3)2? 2Cu(NO3)2+4KI ⟶ 2CuI+I2+4KNO3

161.40 mL KI solution

The toxic pigment called white lead, Pb3(OH)2(CO3)2, has been replaced in white paints by rutile, TiO2. How much rutile (g) can be prepared from 379 g of an ore that contains 88.3% ilmenite (FeTiO3) by mass? 2FeTiO3+4HCl+Cl2 ⟶ 2FeCl3+2TiO2+2H2O

176 g TiO2

The label on a soft drink bottle gives the volume in two units: 2.0 L and 67.6 fl oz. Use this information to derive a conversion factor between the English and metric units. How many significant figures can you justify in your conversion factor?

2.0 L / 67.6 fl oz = 0.030 L / 1 fl oz Only two significant figures are justified.

A cylinder of O2(g) used in breathing by patients with emphysema has a volume of 3.00 L at a pressure of 10.0 atm. If the temperature of the cylinder is 28.0 °C, what mass of oxygen is in the cylinder?

38.8 g

A balloon filled with helium gas takes 6 hours to deflate to 50% of its original volume. How long will it take for an identical balloon filled with the same volume of hydrogen gas (instead of helium) to decrease its volume by 50%?

4.2 hours

Calculate the density of Freon 12, CF2Cl2, at 30.0 °C and 0.954 atm.

4.64 g L^-1

Calculate the volume of oxygen required to burn 12.00 L of ethane gas, C2H6, to produce carbon dioxide and water, if the volumes of C2H6 and O2 are measured under the same conditions of temperature and pressure.

42.00 L

Many medical laboratory tests are run using 5.0 μL blood serum. What is this volume in milliliters?

5.0 × 10−3 mL

Urea, CO(NH2)2, is manufactured on a large scale for use in producing urea-formaldehyde plastics and as a fertilizer. What is the maximum mass of urea that can be manufactured from the CO2 produced by combustion of 1.00×103 kg of carbon followed by the reaction? CO2(g)+2NH3(g) ⟶ CO(NH2)2(s)+H2O(l)

5.00 × 103 kg

Ethanol, C2H5OH, is produced industrially from ethylene, C2H4, by the following sequence of reactions: 3C2H4+2H2SO4 ⟶ C2H5HSO4+ (C2H5)2SO4; C2H5HSO4+ (C2H5)2SO4+3H2O ⟶ 3C2H5OH+2H2SO4 What volume of ethylene at STP is required to produce 1.000 metric ton (1000 kg) of ethanol if the overall yield of ethanol is 90.1%?

5.40 × 10^5 L

When two cotton plugs, one moistened with ammonia and the other with hydrochloric acid, are simultaneously inserted into opposite ends of a glass tube that is 87.0 cm long, a white ring of NH4Cl forms where gaseous NH3 and gaseous HCl first come into contact. NH3(g)+HCl(g) ⟶ NH4Cl(s) At approximately what distance from the ammonia moistened plug does this occur? (Hint: Calculate the rates of diffusion for both NH3 and HCl, and find out how much faster NH3 diffuses than HCl.)

51.7 cm

Convert the boiling temperature of gold, 2966 °C, into degrees Fahrenheit and kelvin.

5371 °F, 3239 K

A cylinder of medical oxygen has a volume of 35.4 L, and contains O2 at a pressure of 151 atm and a temperature of 25 °C. What volume of O2 does this correspond to at normal body conditions, that is, 1 atm and 37 °C?

5561 L

Rebox reaction

Another term for oxidation reduction reaction

How do you calculate the number of neutrons?

A - Z = number of neutrons

Oxidant

Another term for oxidizing agent

Reductant

Another term for reducing agent

Physical change

A change in the state or properties of matter without any accompanying change in the chemical identities of the substances contained in the matter

Significant digits

A significant figures

Meter

A standard metric and SI unit of length; 1 m = approximately 1.094 yards

Hypothesis

A tentative explanation of observations that acts as a guide for gathering and checking information

Lanthanide

A transition metal in the first row of the two bottoms

Actinide

A transition metal in the second row of the two bottoms

Trailing

A zero behind the last significant figure in a number

Captive

A zero between significant figures

Leading

A zeroes between the decimal and the first significant figure

Acidic properties

Ability to release H+ ions when dissolved in water

Is a meter about an inch, a foot, a yard, or a mile?

About a yard

Strong acid

Acid that reacts completely when dissolved in water to yield hydronium ions

Weak acid

Acid that reacts only to a slight extent when dissolved in water to yield hydronium ions

Determine which of the following contains the greatest mass of aluminum: 122 g of AlPO4, 266 g of Al2C16, or 225 g of Al2S3.

AlPO4: 1.000 mol, or 26.98 g Al; Al2Cl6: 1.994 mol, or 53.74 g Al; Al2S3: 3.00 mol, or 80.94 g Al; The Al2S3 sample thus contains the greatest mass of Al

Significant figures

All of the measured digits in a determination, including the uncertain last digit

Law of constant composition

All samples of a pure compound contain the same elements in the same proportions by mass

Dalton

Alternative unit equivalent to the atomic mass unit

Unified atomic mass unit (u)

Alternative unit equivalent to the atomic mass unit

What is Amontons's law

Amontons's law: P/T = constant at constant V and n; If the temperature is increased, the average speed and kinetic energy of the gas molecules increase; If the volume is held constant, the increased speed of the gas molecules results in more frequent and more forceful collisions with the walls of the container, therefore increasing the pressure

Rate of diffusion

Amount of gas diffusing through a given area over a given time

Actual yield

Amount of product formed in a reaction

Theoretical yield

Amount of product that may be produced from a given amount of reactant(s) according to the reaction stoichiometry

What are the parts of an atom?

An atom consists of a small, positively charged nucleus surrounded by electrons

What does it mean to say an equation is balanced? Why is it important for an equation to be balanced?

An equation is balanced when the same number of each element is represented on the reactant and product sides. Equations must be balanced to accurately reflect the law of conservation of matter.

Half-reaction

An equation that shows whether each reactant loses or gains electrons in a reaction; Write the two half reactions representing the redox process; Balance all elements except oxygen and hydrogen; Balance oxygen atoms by adding H2O molecules; Balance hydrogen atoms by adding H+ ions; Balance charge by adding electrons; If necessary, multiply each half reaction's coefficients by the smallest possible integers to yield equal numbers of electrons in each Add the balanced half reactions together and simplify by removing species that appear on both sides of the equation; For reactions occurring in basic media (excess hydroxide ions), carry out these additional steps (Add OH- ions to both sides of the equation in numbers equal to the numbers of H+ ions; On the side of the equation containing both H+ and OH- ions, combine these ions to yield water molecules; Simplify the equation by removing any redundant water molecules); Finally, check to see that both the number of atoms and the total charges are balanced

Gay Lussac's law

Another term for Amontons's Law

Molecular Compound

Another term for covalent compound

Solution

Another term for homogeneous mixture

Noble gas

Another term for inert gas

Law of definite proportions

Another term for law of constant composition

Representative element

Another term for main group element

Covalent compound

Another term for molecular compound

Many of the items you purchase are mixtures of pure compounds. Select three of these commercial products and prepare a list of the ingredients that are pure compounds.

Answers will vary. Sample answer: Gatorade contains water, sugar, dextrose, citric acid, salt, sodium chloride, monopotassium phosphate, and sucrose acetate isobutyrate.

Matter

Anything that occupies space and has mass

Aqueous hydrogen fluoride (hydrofluoric acid) is used to etch glass and to analyze minerals for their silicon content. Hydrogen fluoride will also react with sand (silicon dioxide). (a) Write an equation for the reaction of solid silicon dioxide with hydrofluoric acid to yield gaseous silicon tetrafluoride and liquid water. (b) The mineral fluorite (calcium fluoride) occurs extensively in Illinois. Solid calcium fluoride can also be prepared by the reaction of aqueous solutions of calcium chloride and sodium fluoride, yielding aqueous sodium chloride as the other product. Write complete and net ionic equations for this reaction

Aqueous hydrogen fluoride (hydrofluoric acid) is used to etch glass and to analyze minerals for their silicon content. Hydrogen fluoride will also react with sand (silicon dioxide). (a) Write an equation for the reaction of solid silicon dioxide with hydrofluoric acid to yield gaseous silicon tetrafluoride and liquid water. (b) The mineral fluorite (calcium fluoride) occurs extensively in Illinois. Solid calcium fluoride can also be prepared by the reaction of aqueous solutions of calcium chloride and sodium fluoride, yielding aqueous sodium chloride as the other product. Write complete and net ionic equations for this reaction)

How do you calculate the number of protons?

Atomic number (Z) = number of protons

Mole

Atoms 6.022 * 1023

Isotopes

Atoms that contain the same number of protons but different numbers of neutrons

Covalent bond

Attractive force between the nuclei of a molecule's atoms and pairs of electrons between the atoms

Mean free path

Average distance a molecule travels between collisions

Atomic mass

Average mass of atoms of an element, expressed in amu

Acetate

CH3COO-

Barium

Ba+2

Weak base

Base that reacts only to a slight extent when dissolved in water to yield hydroxide ions

How are protons and neutrons similar? How are they different?

Both are subatomic particles that reside in an atom's nucleus. Both have approximately the same mass. Protons are positively charged, whereas neutrons are uncharged.

Bromide

Br-

Nomenclature

System of rules for naming objects of interest

A major textile dye manufacturer developed a new yellow dye. The dye has a percent composition of 75.95% C, 17.72% N, and 6.33% H by mass with a molar mass of about 240 g/mol. Determine the molecular formula of the dye.

C15H15N3

How do you calculate dilutions

C1V1 = C2V2

Oxalate

C2O4 -2

A compound of carbon and hydrogen contains 92.3% C and has a molar mass of 78.1 g/mol. What is its molecular formula?

C6H6

Cadmium

Cd+2

How to convert celsius to fahrenheit

Celsius = (5/9) * (Fahrenheit - 32)

How to convert celsius to kelvin

Celsius = Kelvin - 273.15

What did Chadwick discover?

Chadwick discovered that the nucleus also contains neutral particles called neutrons

Physical property

Characteristics of matter that is not associated with a change in its chemical composition

Properties

Characteristics that distinguish one substance from another

Complete ionic equation

Chemical equation in which all dissolved ionic reactants and products, including spectator ions are explicitly represented by formulas for their dissociated ions

Molecular equation

Chemical equation in which all reactants and products are represented as neutral substances

Net ionic equation

Chemical equation in which only those dissolved ionic reactants and products that undergo a chemical or physical change are represented

Balanced equation

Chemical equation with equal numbers of atoms for each element in the reactant and product

Analyte

Chemical species of interest

Chlorate

ClO3 -

The pressure of the atmosphere on the surface of the planet Venus is about 88.8 atm. Compare that pressure in psi to the normal pressure on earth at sea level in psi.

Earth: 14.7 lb in^-2; Venus: 1.31 × 10^3 lb in^−2

Ion

Electrically charged atom or molecule; Contains unequal numbers of protons and electrons; can be monatomic or polyatomic

Ionic bond

Electrostatic forces of attraction between the oppositely charged ions of an ionic compound

Representative element

Element in columns 1, 2, and 12-18

Alkali metal

Element in group 1

Pnictogen

Element in group 15

Chalcogen

Element in group 16

Halogen

Element in group 17

Inert gas

Element in group 18

Noble gas

Element in group 18

Alkaline earth metal

Element in group 2

Main group element

Element in groups 1, 2, and 13-18

Transition metal

Element in groups 3-12; Or more strictly defined as 3-11

Inner transition metal

Element in the bottom two rows

Nonmetal

Element that appears dull, poor conductor of heat and electricity; can gain the number of electrons needed to provide atoms with the same number of electrons as in the next noble gas in the periodic table

Metalloid

Element that conducts heat and electricity moderately well, and possesses some properties of metals and some properties of nonmetals

Which of the following gases diffuse more slowly than oxygen? F2, Ne, N2O, C2H2, NO, Cl2, H2S

F2, N2O, Cl2, H2S

How to convert fahrenheit to celsius

Fahrenheit = (9/5 * Celsius) + 32

Liquid

Flows and takes the shape of container, except that it forms a flat or slightly curved upper surface when acted on by gravity (will take the volume in zero g)

For a given amount of gas showing ideal behavior, draw labeled graphs of: (a) the variation of P with V (b) the variation of V with T (c) the variation of P with T (d) the variation of 1 P with V

For a gas exhibiting ideal behavior:

Pressure

Force exerted per unit area

Ideal gas

Hypothetical gas whose physical properties are perfectly described by the gas laws

How are the molecules in oxygen gas, the molecules in hydrogen gas, and water molecules similar? How do they differ?

In each case, a molecule consists of two or more combined atoms. They differ in that the types of atoms change from one substance to the next.

Actinide

Inner transition metal in the bottom two rows of the periodic table

Lanthanide

Inner transition metal in the top of the bottom two rows of the periodic table

Precipitate

Insoluble product that forms from reaction of soluble reactants

Scientific method

Investigating natural phenomenon through the question and observation to create laws or hypothesis which form theories; Hypothesis are then tested through experiments and theories are adjusted

Monatomic ion

Ion composed of a single atom

Polyatomic ion

Ion composed of more than one atom

Periodic table

Table of elements that places elements with similar properties close together

Potassium

K+

How to convert kelvin to celsius

Kelvin = Celsius + 273.15

Macroscopic domain

Larger, observable and measurable physical and chemical properties such as density, solubility, and flammability

Lithium

Li+

What properties distinguish solids from liquids? Liquids from gases? Solids from gases?

Liquids can change their shape (flow); solids can't. Gases can undergo large volume changes as pressure changes; liquids do not. Gases flow and change volume; solids do not.

Example of derived units

Liters (density and volume

Why should you roll or belly-crawl rather than walk across a thinly-frozen pond?

Lying down distributes your weight over a larger surface area, exerting less pressure on the ice compared to standing up. If you exert less pressure, you are less likely to break through thin ice.

How do you calculate molarity

M = mol solute / L solution

The amount of heat required to melt 2 lbs of ice is twice the amount of heat required to melt 1 lb of ice. Is this observation a macroscopic or microscopic description of chemical behavior? Explain your answer.

Macroscopic. The heat required is determined from macroscopic properties

Molar mass

Mass in grams of 1 mole of a substance

How do you calculate the mass number?

Mass number (A) = number of protons + number of neutrons

Nucleus

Massive, positively charged center of an atom made up of protons and neutrons

What is Dalton's atomic theory

Matter is composed of exceedingly small particles called atoms; An element consists of only one type of atom, which has a mass that is characteristic of the element and is the same for all atoms of that element; Atoms of one element differ in properties from atoms of all other elements; A compound consists of atoms of two or more elements combined in a small, whole number ratio; Atoms are neither created nor destroyed during a chemical change, but are instead rearranged to yield substances that are different from those present before the change

Pure substance

Matter with a constant composition

Root mean square velocity (urms)

Measure of average velocity for a group of particles calculated as the square root of the average squared velocity

Length

Measure of one dimension of an object

Mass

Measure of the amount of matter in an object

Volume

Measure of the amount of space occupied by an object

Percent yield

Measure of the efficiency of a reaction, expressed as a percentage of the theoretical yield

Symbolic domain

Specialized language used to represent components of the macroscopic and microscopic domains; Chemical symbols, chemical formulas, and chemical equations; Help to interpret the behavior of the macroscopic domain in terms of the components of the microscopic domain

Strontium

Sr+2

Kilogram

Standard SI unit of mass; 1 kg = approximately 2.2 pounds

Units

Standards of comparison for measurements

Indicator

Substance added to the sample in a titration analysis to permit visual detection of the end point

Product

Substance formed by a chemical or physical change; Shown on the right side of the arrow in a chemical equation

Oxidizing agent

Substance that brings about the oxidation of another substance, and in the process becomes reduced

Reducing agent

Substance that brings about the reduction of another substance, and in the process becomes oxidized

Acid

Substance that produces H3O+ when dissolved in water

Base

Substance that produces OH- when dissolved in water

Reactant

Substance undergoing a chemical or physical change; Shown on the left side of the arrow in a chemical equation

Empirical formula mass

Sum of average atomic masses for all atoms represented in an empirical formula; (AxBy)n = AnxBny

Formula mass

Sum of the average masses for all atoms represented in a chemical formula; For covalent compounds, this is also the molecular mass

Mass number (A)

Sum of the numbers of neutrons and protons in the nucleus of an atom; approximately equal to the mass of the atom

Law

Summarize a vast number of experimental observations, and describe or predict some facet of the natural world

Chemical equation

Symbolic representation of a chemical reaction

A 0.025-g sample of a compound composed of boron and hydrogen, with a molecular mass of ~28 amu, burns spontaneously when exposed to air, producing 0.063 g of B2O3. What are the empirical and molecular formulas of the compound?

The empirical formula is BH3. The molecular formula is B2H6.

The reaction of WCl6 with Al at ~400 °C gives black crystals of a compound containing only tungsten and chlorine. A sample of this compound, when reduced with hydrogen, gives 0.2232 g of tungsten metal and hydrogen chloride, which is absorbed in water. Titration of the hydrochloric acid thus produced requires 46.2 mL of 0.1051 M NaOH to reach the end point. What is the empirical formula of the black tungsten chloride?

The empirical formula is WCl4

Solubility

The extent to which a substance may be dissolved in water, or any solvent

Under which of the following sets of conditions does a real gas behave most like an ideal gas, and for which conditions is a real gas expected to deviate from ideal behavior? Explain. (a) high pressure, small volume (b) high temperature, low pressure (c) low temperature, high pressure

The gas behavior most like an ideal gas will occur under the conditions in (b). Molecules have high speeds and move through greater distances between collision; they also have shorter contact times and interactions are less likely. Deviations occur with the conditions described in (a) and (c). Under conditions of (a), some gases may liquefy. Under conditions of (c), most gases will liquefy.

International system of units

The international standard measurement units; Also called SI units

What is the limiting reactant in a reaction that produces sodium chloride from 8 g of sodium and 8 g of diatomic chlorine?

The limiting reactant is Cl2.

How does a homogeneous mixture differ from a pure substance? How are they similar?

The mixture can have a variety of compositions; a pure substance has a definite composition. Both have the same composition from point to point.

Fundamental aspects of chemical equations

The substances undergoing reaction are called reactants, and their formulas are placed on the left side of the equation; The substances generated by the reaction are called products, and their formulas are placed on the right side of the equation; Plus signs (+) separate individual reactant and product formulas, and arrow separates the reactant and product (left and right) sides of the equation; The relative numbers of reactant and product species are represented by coefficients

Explain why the symbol for an atom of the element oxygen and the formula for a molecule of oxygen differ.

The symbol for the element oxygen, O, represents both the element and one atom of oxygen. A molecule of oxygen, O2, contains two oxygen atoms; the subscript 2 in the formula must be used to distinguish the diatomic molecule from two single oxygen atoms.

In the following drawing, the green spheres represent atoms of a certain element. The purple spheres represent atoms of another element. If the spheres of different elements touch, they are part of a single unit of a compound. The following chemical change represented by these spheres may violate one of the ideas of Dalton's atomic theory. Which one?

The starting materials consist of one green sphere and two purple spheres. The products consist of two green spheres and two purple spheres. This violates Dalton's postulate that that atoms are not created during a chemical change, but are merely redistributed

Chemistry

The study of the composition, properties, and interactions of matter

10 atoms

deca-

Freon-12, CCl2F2, is prepared from CCl4 by reaction with HF. The other product of this reaction is HCl. Outline the steps needed to determine the percent yield of a reaction that produces 12.5 g of CCl2F2 from 32.9 g of CCl4. Freon-12 has been banned and is no longer used as a refrigerant because it catalyzes the decomposition of ozone and has a very long lifetime in the atmosphere. Determine the percent yield.

gCCl4 ⟶ molCCl4 ⟶ molCCl2F2 ⟶ gCCl2F2, percent yield=48.3%

7 atoms

hepta-

6 atoms

hexa-

4 atoms

tetra


Related study sets

Chapter 3 - Proteins (Practice Quiz)

View Set

Chapter 12: Social Psychology (Part 2)

View Set

Cognitive Psychology Chapter 6 & 7

View Set

BIOLOGY 346 Microbes and society

View Set

The Cell - Anatomy and Division Lab Assignment

View Set

fahmy 1 ( bible ) Arabic & German 7

View Set